Rings, ideals, prime and maximal

In summary, the conversation discusses a proof for the statement that the set R \setminus \{ \cup_{i \in \Lambda} P_{i} \} is a multiplicative system, where \{P_{i} : i \in \Lambda \} is a family of prime ideals in a ring R. The conversation also mentions relevant theorems and raises concerns about the validity of the proof. The expert summarizer concludes by providing a summary of the proof and stating that the conversation does not require any theorems, but only the definition of a prime ideal.
  • #1
futurebird
272
0
I just turned in this homework and I want to know if I got it right. The proof is pretty simple, but I think I might be using a theorem in the wrong way.

Homework Statement



[tex]\{P_{i} : i \in \Lambda \}[/tex] is a family of prime ideals in a ring, R. Prove that [tex]R \setminus \{ \cup_{i \in \Lambda} P_{i} \}[/tex] is a multiplicative system.


2. Relevant theorems

  1. Let R be a ring. An element [tex]u \in R[/tex] is called a unit if it has an inverse in R.
  2. Let R be a ring. The union of all maximal ideals of R is the set of non-units in R.
  3. Any Maximal ideal is also a prime ideal .

The Attempt at a Solution


Any maximal ideal is also a prime ideal. Hence,
[tex]\cup_{M max} M \subseteq \cup_{i \in \Lambda} P_{i}[/tex].
Then every element in
[tex]R \setminus \{ \cup_{i \in \Lambda} P_{i} \}[/tex]
is a unit. (We have removed all non-units and, possibly, some units since there may be prime ideals that are not maximal.) The set of units is closed under multiplication because, if a and b are units, then c=ab has an inverse [tex]c^{-1}=b^{-1}a^{-1}[/tex].
[tex]1 \in R \setminus \{ \cup_{i \in \Lambda} P_{i} \}[/tex].
1 is not in any of the prime ideals, hence,
[tex]R \setminus \{ \cup_{i \in \Lambda} P_{i} \}[/tex]
is a multiplicative system.



So would this work? It bothers me that when we remove [tex] \cup_{i \in \Lambda} P_{i} [/tex] from R we are not just taking out the maximal ideals, but also the prime ideals that may not be maximal. Could this lead to some non-units remaining in the set?
 
Physics news on Phys.org
  • #2
What is a multiplicative system? A multiplicative group?
 
  • #3
morphism said:
What is a multiplicative system? A multiplicative group?

A multiplicative system is a set closed under multiplication and containing the multiplicative identity.
 
  • #4
bump.
 
  • #5
Then I don't think your proof is valid as it stands.
 
  • #6
morphism said:
Then I don't think your proof is valid as it stands.

Because of the issue I mentioned or something else.
 
  • #7
Because of the issue you mentioned.
 
  • #8
This doesn't require any theorems, really. Only the definition of a prime ideal. Just take an arbitrary product of elements in the set, ie. elements that are NOT in any of these prime ideals. Hence if the set were not multiplicatively closed, this product would be in the union of the prime ideals and hence contained in some prime ideal (by definition of a set theoretic union) and hence, by definition of prime ideal, some element would be in the prime ideal, contradicting the element NOT being in any of the prime ideals.
 

Related to Rings, ideals, prime and maximal

1. What is a ring in mathematics?

A ring is an algebraic structure consisting of a set of elements with two binary operations, usually addition and multiplication. These operations must follow certain rules, such as closure, associativity, and distributivity, to be considered a ring.

2. What is an ideal in a ring?

An ideal is a subset of a ring that is closed under addition, subtraction, and multiplication by elements of the ring. It is a generalization of the concept of divisibility in integers.

3. What is a prime ideal in a ring?

A prime ideal is an ideal that is also a prime element in the ring. This means that it is not the entire ring, and if two elements in the ring multiply to give an element in the prime ideal, then at least one of the original elements must also be in the prime ideal.

4. What is a maximal ideal in a ring?

A maximal ideal is an ideal that is not a proper subset of any other ideal in the ring. In other words, it is a "biggest" ideal in the ring, and any other ideal that contains it must be the whole ring.

5. How are prime and maximal ideals related?

In a commutative ring, a maximal ideal is always a prime ideal. This is because in a commutative ring, the quotient ring obtained by dividing by a maximal ideal is always a field, and fields have no non-trivial ideals. However, in a non-commutative ring, this relationship does not hold true.

Similar threads

  • Calculus and Beyond Homework Help
Replies
14
Views
2K
  • Calculus and Beyond Homework Help
Replies
5
Views
2K
  • Calculus and Beyond Homework Help
Replies
1
Views
981
  • Calculus and Beyond Homework Help
Replies
3
Views
2K
  • Calculus and Beyond Homework Help
Replies
2
Views
2K
  • Calculus and Beyond Homework Help
Replies
1
Views
706
  • Calculus and Beyond Homework Help
Replies
1
Views
861
  • Calculus and Beyond Homework Help
Replies
14
Views
551
  • Calculus and Beyond Homework Help
Replies
1
Views
564
  • Advanced Physics Homework Help
Replies
1
Views
1K
Back
Top